he temperature at noon at a ski resort is 3°F. The temperature drops 8°F by midnight.
What is the temperature at midnight?

Answers

Answer 1

Answer:

thye answer is -5f

Step-by-step explanation


Related Questions

What is the simplified value of the expression below? StartFraction 4 (negative 8 + 4.5) over 6.25 + (negative 8.25) EndFraction –25 –7 7 25

Answers

Answer: -10.49.

Step-by-step explanation:

Using order of operations PEMDAS:

4(-8+4.5)/ 6.25 + (-8.25)

= 4 * -3.5 / 6.25 - 8.25

= -14/6.25 - 8.25

=  -2.24 - 8.25

= -10.49.

Answer:

The answer should be -10.49

Step-by-step explanation:

Using the distributive property, which of the following is an equivalent expression for 3(9 + 15)

Answers

Answer:

3 x 9+3 x 15

Step-by-step explanation:

Simplify 2(9* 5+3^2) + 4

Answers

Answer:

112

Step-by-step explanation:

Order of Operations: BPEMDAS

Step 1: Write out expression

2(9 × 5 + 3²) + 4

Step 2: Exponents

2(9 × 5 + 9) + 4

Step 3: Multiplication

2(45 + 9) + 4

Step 4: Parenthesis

2(54) + 4

Step 5: Multiply

108 + 4

Step 6: Add

112

the answer would be 112
i think

10 10 times as much as what

Answers

Answer:

100

Step-by-step explanation:

10x10

10 times 10 is equal to 100.

E
OV
Question 3/18
<
> NEXT
14
BOOKMARK
3
Mrs. Scarbrough and her dog drove 204 miles to grandma's house. The trip took them 3 hours.
What was their average speed in miles per hour?
miles per hour

Answers

the average speed is 68 m/hr

Find the general solution of the given system.
dx dt = 6x − y
dy dt = 36x − 6y

Answers

Answer:

[tex]y = 6x + C[/tex]

Step-by-step explanation:

Given the system of equations

dx/dt = 6x − y  ............. 1

dy/dt = 36x − 6y ............ 2

Divide equation 2 by 1;

[tex]\dfrac{dy/dt}{dx/dt} = \dfrac{36x-6y}{6x-y} \\\\\dfrac{dy/dt}{dx/dt} = \dfrac{6(6x-y)}{6x-y}\\ \\\dfrac{dy/dt}{dx/dt} = 6\\\\\dfrac{dy}{dt}* \dfrac{dt}{dx} = 6\\\dfrac{dy}{dx} = 6[/tex]

cross multiply

[tex]dy = 6dx\\[/tex]

integrate both sides of the expression

[tex]\int\limits {dy} = \int\limits {6} \, dx \\\\y = 6x + C[/tex]

Hence the general solution to the system of equation is [tex]y = 6x + C[/tex]

which statement is false A: every irrational number is also a real number B: every integer is also a real number C: every integer is also an irrational number D: no irrational number is rational

Answers

Answer is D

I say this because I can't be a cause international numbers real

B every integers a real number

C every integers is also an interration number.

The definition of a circle uses the underfined term

Arc.
Line.
Plane
Ray

Answers

Arc because a circle is round
Arc because the circle is round

The total weight of a rock depends on its size and is proportional to its density. In this context, density is the weight per cubic inch. Let w denote the weight of the rock in pounds, s the size of the rock in cubic inches, and d the density of the rock in pounds per cubic inch. If a 48-cubic-inch rock weighs w pounds, write an equation that shows the proportional relation.

Answers

Answer:

[tex]d = \frac{w}{48}[/tex]

If  [tex]\frac{1}{48} \\[/tex] is constant, say k

Then,

[tex]d = kw[/tex]

∴ d ∝ w

Hence, weight is proportional to the density

Step-by-step explanation:

From the question,

Let w denote the weight of the rock in pounds

s denote the size of the rock in cubic inches and

d denote the density of the rock in pounds per cubic inch.

First, we will write the equation connecting w, s, and d.

We get

[tex]density (pounds/inch^{3} ) = \frac{weight(pounds)}{size (inch^{3}) }[/tex]

That is,

[tex]d = \frac{w}{s}[/tex]

Now, given a 48-cubic-inch rock with weight w pounds, to show the proportional relation between the weight and the density, we will write

[tex]d = \frac{w}{48}[/tex]

If  [tex]\frac{1}{48} \\[/tex] is constant, say k

Then,

[tex]d = kw[/tex]

∴ d ∝ w

Hence, density is proportional to the weight OR weight is proportional to the density

I already know the answer but Brainliest for best explanation 3973-2937

Answers

Answer:

1036

Step-by-step explanation:

Working each column from right to left

 

3 9 7 3

− 2 9 3 7

 

7 is greater than 3 so you must regroup:

Take 1 from 7, so 7 becomes 6.

Add 10 to 3, so 3 becomes 13.

 

6  13

 3 9 --

− 2 9 3 7

 

13 minus 7 is 6

 

6  13

 3 9 --

− 2 9 3 7

 

6

6 minus 3 is 3

 

6 13

 3 9 --

− 2 9 3 7

 3 6

9 minus 9 is 0

 

6  13

 3 9 --

− 2 9 3 7

 0 3 6

3 minus 2 is 1

 

6  13

 3 9 --

− 2 9 3 7

 1 0 3 6

Hope this helps! Plz award Brainliest : )

Answer:

1036

Step-by-step explanation:

If Jonny and Suzy share a $10 burger, how much does each owe?

Answers

Answer: $5

Step-by-step explanation:

10/2=5

2 people x $10 = $5

65.8 percent of what number is 14

Answers

Answer:

[tex] \boxed{ \bold{ \huge{ \boxed{ \sf{21.27}}}}}[/tex]

Step-by-step explanation:

Let the number be x

Create an equation

[tex] \sf{65.8\% \: of \: x = 14}[/tex]

[tex] \longrightarrow{ \sf{ \frac{65.8}{100} x = 14}}[/tex]

[tex] \longrightarrow{ \sf{x = \frac{14}{ \frac{65.8}{100} } }}[/tex]

[tex] \longrightarrow{ \sf{x = 14 \times \frac{100}{65.8}}} [/tex]

[tex] \longrightarrow{ \sf{x = 21.27}}[/tex]

Hope I helped!

Best regards! :D

On a map, 1 cm represents 300 km. What does 3/5 cm represent?​

Answers

Answer:

180 km

Step-by-step explanation:

3/5 times 300 is 180

Answer

180 km

Step-by-step explanation:

Muna is making a scale drawing of a gecko. The gecko is 1:2 inch wide and 5 inches long. she decides to make her drawing of the gecko 1 inch wide. How long will the drawing be ? show your work

Answers

Answer:

L= 0.83 in

Step-by-step explanation:

Width= 1.2 in when length = 1 in

Required

Determine the length when width = 1 in

First, we need to determine the ratio

Ratio = [tex]\frac{Width}{Length}[/tex]

Ratio = [tex]\frac{1.2}{1}[/tex]

Ratio = 1.2 ---(1)

Let the required length be represented with L

So:

Ratio = [tex]\frac{Width}{Length}[/tex]

Ratio = [tex]\frac{1}{L}[/tex] ---(2)

Equate (1) and (2)

[tex]\frac{1}{L}[/tex] = 1.2

Multiply through by 1.2

L * [tex]\frac{1}{L}[/tex] = 1.2 * L

1 = 1.2 * L

Divide through by 1.2

L = [tex]\frac{1}{1.2}[/tex]

L = 0.83 in (Approximated)

Which are correct representations of the inequality -3(2x - 5) <5(2 - x)? Select two options.
x < 5
0-6x-5 < 10 - X
0-6x + 15 < 10 - 5x
-3 -2 -1
0
1
2
2
3
4
5 6 7
+
口……
-7 -6 -5
-4
-3
-2
-1
0
1
2
3

Answers

Answer:

what is this??

Step-by-step explanation:

I really don't understand this question can someone please explain to me

When the polynomial:
[tex]h(x) ={dx}^{3}+{fx}^{2} - 7x - 6[/tex]
is defined by
[tex]x - 1[/tex]
its remainder is
[tex] - 10[/tex]
and when it is divided by
[tex]x - 3[/tex]
the remainder is
[tex]36[/tex]
find the values of the constant d and f

Answers

Thanks this helps a lot

Math need help fast.............................

Answers

Answer: B) 3 times

Step-by-step explanation:

write an equation

2/3x=2

Muktpily 3/2 on both sides

Youll get 3

What is the answer to 24/3x-2

Answers

Answer:

=8x−2

Step-by-step explanation:

I hope this helps you

Does anyone know if this is right if not which one is it

Answers

Answer:

Step-by-step explanation:

The answer is actually the last one :)

Do the binary numbers “0011” and “000011” have the same value or different values? Explain.

Answers

Using binary to decimal conversion, it is found that the numbers have the same values.

----------------------------------

0011

Converting from decimal, the number is given by:

[tex]0011 = 1\times2^0 + 1\times2^1 + 0\times2^2 + 0\times2^3 = 1 + 1 + 0 + 0 = 2[/tex]

----------------------------------

000011

Converting from decimal, the number is given by:

[tex]000011 = 1\times2^0 + 1\times2^1 + 0\times2^2 + 0\times2^3 + 0\times2^4 + 0\times2^5 = 1 + 1 + 0 + 0 + 0 + 0 = 2[/tex]

In both cases, the result of conversion is 2, thus, they have the same value.

A similar problem is given at https://brainly.com/question/7978210

To transform a binary number to a decimal number we need to multiply each of the components of the binary number by 2 to the power of the place value minus one, and sum all of that.

Based on this, we will see that 0011 and 000011 have the same value.

For example, the number 01 will be:

[tex]0*2^1 + 1*2^0 = 1[/tex]

The number 1010 will be:

[tex]1*2^3 + 0*2^2 + 1*2^1 + 0*2^0 = 1*8 + 1*2 = 10[/tex]

Now let's convert the two given numbers to decimals:

0011

[tex]0*2^3 + 0*2^2 + 1*2^1 + 1*2^0 = 3[/tex]

000011

[tex]0*2^5 + 0*2^4 + 0*2^3 + 0*2^2 + 1*2^1 + 1*2^0 = 3[/tex]

So exactly like in decimal numbers, adding zeros to the left side does not affect the value.

If you want to learn more, you can read:

https://brainly.com/question/19802955

what is 2,340 divided by 36

Answers

Answer:

65.

Step-by-step explanation:

You could honestly just use a calculator for this, but okay.

the admission fee to an amusement park is $17. It costs an additional c dollars to rent a locker to hold your belongings. the total cost for 10 to enter the amusement park and each rent a locker is $260 how much does it cost for one person to rent a locker

Answers

Answer:

$9

Step-by-step explanation:

$260-$170=90

90/10=9

We know that $17 * 10 = $170

c = amount of money to rent a locker.

$170 + 10c = $260

Subtract $170 from both sides.

10c = $90

Divide both sides by 10

c = $9

It cost $9 for one person to rent a locker.

Sleeping in college. A recent article in a college newspaper stated that college students get an average of 5.5 hrs of sleep each night. A student who was skeptical about this value decided to conduct a survey by randomly sampling 25 students. On average, the sampled students slept 6.25 hours per night. Identify which value represents the sample mean and which value represents the claimed population mean.

Answers

Answer:

Sample mean = 6.25 hours per night.

Population mean = 5.5 hrs of sleep each night.

Step-by-step explanation:

A sample mean is the mean of the sample collected. The 25 students surveyed by the student is the sample. The average sleep time derived from the sample is the sample mean.

A population mean is the mean of all the population. Here the population are college students. The population mean is the mean derived from studying the sleep duration of all the population - college students

To prevent illness and Injury, employers must make standardized health and safety Information for the chemicals
they produce avallable to employees for all chemicals on the job and train employees to find this Information. true or false?

Answers

Answer:

true

Step-by-step explanation:

if they wash there hands the customers and employees can be safe

Josh wrote a whole number using the digits 8,4,7,2,9 and 0. Josh's number has a seven in the thousands and a 2 in the tens place. Josh's number is less then 500,000. Is Josh's number greater than 480,000?

Answers

Answer:

Step-by-step explanation:

Using the digits 8,4,7,2,9 and 0 to write a whole Number with 7 in the thousands place and 2 in the tens place :

The whole number is 6 digits long

100 thousand

10 thousand

Thousand = 7

Hundred

Tens = 2

Unit

Also the whole number is less than 500,000

Hence, number in the 100 thousand place is less Than 5,

Hence digit in the 100 thousand place = 4

Placing either 8 or 9 in the ten thousand place will mean That Josh's whole number is greater than 480,000

Please help! I am confused and cannot get the correct answer.

Answers

Answer:

520

Step-by-step explanation:

I used big brain

-

Use the Squeeze Theorem to show that [tex]\lim_{t \to \infty} \frac{cos5t}{5t}[/tex]
Complete the following argument: for t arbitrary large and positive.

Im not sure how to break this problem down using the Squeeze Theorem, could anyone help?

Answers

Recall that [tex]\cos x[/tex] is bounded between -1 and 1 for all real [tex]x[/tex], so that

[tex]-1\le \cos(5t)\le 1[/tex]

[tex]\implies-\dfrac1{5t}\le\dfrac{\cos(5t)}{5t}\le\dfrac1{5t}[/tex]

Now take the limit of each side of the inequality:

[tex]\displaystyle\lim_{t\to\infty}\left(-\dfrac1{5t}\right)\le\lim_{t\to\infty}\dfrac{\cos(5t)}{5t}\le\lim_{t\to\infty}\dfrac1{5t}[/tex]

The bounding limits are both 0, so by the squeeze theorem the desired limit is also 0.

What is this called?


MATH

Answer and I will give you brainiliest

Answers

Answer:

injective function where range can't have many domain

Step-by-step explanation:

Get off this app u is too young

Crystal Rollerbladed 150 feet in 15 seconds, while Gavin rollerbladed 150 feet in 10 seconds. Without doing any calculations, who is faster? Why?

Crystal was faster because her distance was less than Gavin's.
Crystal was faster because her time was less than Gavin's.
Gavin was faster because his distance was less than Crystal's.
Gavin was faster because his distance was less than Crystal's.

Answers

Answer: Gavin was faster because his time was less than Crystal's.

Step-by-step explanation: They rollerbladed the same distance but Gavin did it in less time.

Hope this helped!

Mark Brainliest if you want!

Answer:

i belive the answer is D tell me if im doing it wrong

Step-by-step explanation:

Use the figure to write the segment addition postulate, write an equation and solve for x.

Answers

Answer:

5). x = -1

6). x = 2

Step-by-step explanation:

5). AC = (x + 10)

    AB = 5

    BC = (2x + 6)

    By segment addition postulate,

    AC = AB + BC

    (x + 10) = 5 + (2x + 6)

    x + 10 = 2x + 11

    2x - x = 10 - 11

    x = -1

6). AC = 19

    AB = 9x + 7

    BC = -3x + 20

    By segment addition postulate,

    AC = AB + BC

    39 = 9x + 7 + (-3x + 20)

    39 = (9x - 3x) + (7 + 20)

    39 = 6x + 27

    6x = 39 - 27

    6x = 12

    x = 2

    x = 2

Other Questions
The centroid of the plane region shown is at c. use the method of composite area to determine the radius (R)? please someone help me i nedd your helps guys please! When elements gain electrons (negative friends), they become more ________A. positiveB. neutralC. negative which art movement did Walter Gropius start that emphasized simple designs using basic materials? Jackson says he has an odd number of models cars. He has 6 cars on one shelf on one shelf and 8 cars on another shelf.is Jackson correct?explain. Is It in Equilibrium?Canvas Will crown first answer brainliest!!!1) Half a number decreased by ten equals thirty. What is the number? should we make a box for report writing THE TASK: Re-write this letter, correctly understanding its purpose, by following the writing process and the 7 Cs, you have studied during the course. Please use the letter format. You can make necessary assumptions and delete/add information. I am writing to inquire about your recent order for a custom wedding suit. You forgot to mention what color you want, and you failed to include your measurements. I would like to clear up this confusion quickly so that we will be able to provide you with the suit before the wedding. When, exactly, is the happy day? I know you must be busy getting ready for the wedding, but if you can spare the time, you might want to stop by in person to pick out your suit, because we do offer an incredibly wide selection of fine wedding attire. At that time, you could also select an appropriate tie and shirt. I would also suggest that you choose clothing for your best man and ushers, assuming that you are having a large wedding. We are prepared to provide the best in both custom and rental formal attire for your entire wedding party, regardless of how large or small it may be. Incidentally, you should also bring your fiance along to coordinate the mens clothing with the bridesmaids dresses. You know how picky women are about clothes. If you cant come by in person, you should send me a letter stating your measurements and indicating your color and style preferences, or call me at 633-4296. After we receive this information, we will need at least two weeks to complete the suit. Thank you for your cooperation in this matter. Frozen foods tend to be high in?A. fat and sodium.B. cost per servingC. essential mineralsD. nutrients and vitamins For all the problems in this assignment you must enter your answers as a fraction or a product of fractions. For example, if your answer is 4/52 * 3/51 or 1 - 13/52 * 12/51, leave it in that form using * to indicate multiplication and ^ to indicate raising a number to a certain power. (eg. (5/6)^3 means 5/6*5/6*5/6 or (5/6)3.) a. Draw 2 cards with replacement. What is the chance that the first is a heart and the second is a heart Andrea's opponent was honest and straightforward, but Andrea used her guile to win the contest. What type of context clue in the sentence can be used to determine the meaning of guile? a definition a synonym an antonym an inference find the midpoint of pine DE if D(-5,-2) and E(7,-5) Which of the following vegetables provides the largest amount of roughage in the diet?a. beansb. cabbagec. carrotd. pepper Work your brain today y'all!! I had $3.00. My Mom gave me $10.00. My Dad Gave me $30.00. My Aunt & Uncle gave me $100.00. I had another $7.00. How much did I have? Right answer will get deleted. I won against: Kelli Kimber How would you describe Red Chief Can someone read my translation over and tell me if I made any errors. Please don't just use google translate. Thank You.Hola, mi nombre es Abhi. Voy a hablar de tres de mis amigos. Son Samit, Jake y Ryan. Todos nos llevamos bien y nos divertimos. Samit es inteligente y va a una escuela diferente. Jake es alto y divertido. Ryan juega al ftbol y es atltico. Todos jugamos videojuegos en mi casa. I liked the movie about the national spelling bee champion. linking verb The booster club needs to raise at least $9,000 for new football uniforms. So far, they have raised $1,220. Enter an inequality to find the average amounts each of the 98 members can raise to meet the club's objective. help please? like anyone Why do some stars become red supergiants?